The author of passage B would be most likely to regard the conclusion that the Solnick and Hemenway study points to t...

Vennela on August 18, 2020

Why is A right and B wrong?

the passage B says that the explanation is not what is BEST supported by the evidence; can't that be taken to mean that it's not ideal but it somewhat makes sense? therefore, weakly supported?

Reply
Create a free account to read and take part in forum discussions.

Already have an account? log in

Shunhe on August 19, 2020

Hi @Vennela-Vellanki,

Thanks for the question! So we’re being asked here for what the author of passage B thinks about the conclusion that the Solnick and Hemenway study points to as the existence of a “phenomena of rivalry” in line 24. Well, they gave us a line, so we should go to it and read above and below for context. And what does the author of passage B think about that line in passage A? Well, the author of passage B disagrees with it, right? The author of passage B says that it’s not a phenomenon of rivalry, but instead, we’re trying to bring value to society.

Now let’s take a look at (B), which says that the author of B thinks that the “phenomena of rivalry” conclusion is flattering in its implications about human nature but only weakly supported by the available evidence. Sure, maybe you think that it is “weakly supported.” But remember, this answer choice has two parts, and both of them need to be correct. Would the author of passage B say that the phenomena of rivalry is “flattering in its implications about human nature?” No, they wouldn’t. It’s not a flattering picture at all, it’s one where people are basically trying to just be better than the people around them! So the first part is off-base, and thus (B) is incorrect.

Now take a look at (A), which says that the theory is ungenerous in its view of human nature and mistaken in its interpretation of the evidence. Well, now we have an answer choice that has both parts correct! It’s not very flattering, or not generous, in its view of human nature. And the author of passage B also thinks it interpreted the evidence incorrectly. So that makes (A) the better answer choice here.

Hope this helps! Feel free to ask any other questions that you might have.